Find f(x) and g(x) so the function can be expressed as y = f(g(x) y = [tex]\frac{8}{x^2}[/tex]+4

Answers

Answer 1

Answer:

Step-by-step explanation:

Hello,

[tex]f(g(x))=\dfrac{8}{x^2}+4[/tex]

So if we take

[tex]f(x)=\dfrac{8}{x}+4 \ \ and \\\\g(x)=x^2\\ \\f(g(x))=\dfrac{8}{g(x)}+4=\dfrac{8}{x^2}+4[/tex]

Hope this helps.

Do not hesitate if you need further explanation.

Thank you


Related Questions

What is the rise over run for the slope -11/9

Answers

Answer:  11 down and 9 right

Step-by-step explanation:

Slope IS rise over run where the top number of the fraction (numerator) determines the vertical distance --> positive is up, negative is down

and the bottom number of the fraction (denominator) determines the horizontal distance --> positive is right, negative is left.

Given slope = -11/9

the numerator is -11 so the "rise" is  DOWN 11 units

the denominator is 9 so the "run" is RIGHT 9 units

help with math ASAP!

Answers

Answer:

1.)  [tex]\frac{1}{9^4}*9^3[/tex]

2.) [tex]\frac{1}{w^7}[/tex]

3.)

Step-by-step explanation:

When you have a negative exponent, rewrite:

[tex]x^{-a}=\frac{1}{x^a}[/tex]

Rewrite using this to change all negative exponents.

Answer:

Multiple Answers

Step-by-step explanation:

Note: When multiplying numbers with exponents, you add the exponents. When dividing numbers with exponents, you subtract exponents.When you have a negative exponent, flip the fraction and write it as a positive exponent.

1) -4 + 3= -1  

So we have  (9^-4) + (9^3)= (1/(9^1)

2) (1/w)^7

3) cannot read problem, but just apply the rules I wrote under "Note"

4) 14/y

5) cannot read problem,but just apply the rules I wrote under "Note"

6) 20d^4 n^? --Cannot read n exponents--.

7) cannot read problem

8) Cannot read problem

9) 90/z^4---only if exponents are 5,-3,and-6

10) 1/(9^5)

11) 54b^4

12) Cannot read problem

13) 16d^8c^8 ---if exponents are 5,3,6,2--

14) s^8

Hope this helps! Plz give brainly, I kinda need it.

How many variables terms are in the expression 3xcube y+5xsquare+y+9

Answers

Answer: Please Give Me Brainliest, Thank You!

2

Step-by-step explanation:

There are two variables here, X and Y

Use the model to show to help find the sum 0.34 plus 0.49

Answers

Answer/Step-by-step explanation:

The idea to use in solving this problem using the model, is to express the number of shaded boxes in fraction form.

Thus, the blue red shaded boxes has 34 boxes shaded out of 100 boxes. This represents [tex] \frac{34}{100} [/tex]. This will give us 0.34.

The other shaded boxes represents [tex] \frac{49}{100} = 0.49 [/tex].

Using the model, we can solve 0.34 + 0.49.

Add both fractions together.

[tex] \frac{34}{100} + \frac{49}{100} = \frac{34+49}{100} [/tex]

[tex] \frac{83}{100} = 0.83 [/tex]

6. If x + 2 is the only factor of the polynomial P(x),then P(2) is:
Options:
A. Cannot be determined
B. Not Zero
C. R(2)
D. Zero

Answers

Answer:

P(x) = x + 2p(2) = 2 + 2 p(2) = 4

So option B is the answer.

If x + 2 is the only factor of the polynomial P(x) then we need to find the P(2) is Not Zero. Therefore, the option B is the correct answer.

What is standard form of a polynomial?

Suppose the considered polynomial is of only one variable.

Then, the standard form of that polynomial is the one in which all the terms with higher exponents are written on left side to those which have lower exponents.

Given information;

If x + 2 is the only factor of the polynomial P(x) then we need to find the P(2) :

P(x) = x + 2

p(2) = 2 + 2

p(2) = 4

The P(2) is Not Zero.

Therefore, the option B is the correct answer.

Learn more about standard form of a polynomial here:

https://brainly.com/question/15313798

#SPJ5

4 Points] Under the HMM generative model, what is p(z1 = z2 = z3), the probability that the same die is used for the first three rolls? b. [4 Points] Suppose that we observe the first two rolls. What is p(z1 = 1 | x1 = 2, x2 = 4), the probability that the casino used the fair die in the first roll?

Answers

Answer:

Step-by-step explanation:

We first examine a simple hidden Markov model (HMM). We observe a sequence of rolls of a four-sided die at an "occasionally dishonest casino", where at time t the observed outcome x_t Element {1, 2, 3, 4}. At each of these times, the casino can be in one of two states z_t Element {1, 2}. When z_t = 1 the casino uses a fair die, while when z_t = 2 the die is biased so that rolling a 1 is more likely. In particular: p (x_t = 1 | z_t = 1) = p (x_t = 2 | z_t = 1) = p (x_t = 3 | z_t = 2) = p (x_t = 4 | z_t = 1) = 0.25, p (X_t = 1 | z_t = 2) = 0.7, p (X_t = 2 | z_t = 2) = p (X_t = 3 | z_t = 2) = p (X_t = 4 | z_t = 2) = 0.1. Assume that the casino has an equal probability of starting in either state at time t = 1, so that p (z1 = 1) = p (z1 = 2) = 0.5. The casino usually uses the same die for multiple iterations, but occasionally switches states according to the following probabilities: p (z_t + 1 = 1 | z_t = 1) = 0.8, p (z_t = 2) = 0.9. The other transition probabilities you will need are the complements of these. a. Under the HMM generative model, what is p (z1 = z2 = z3), the probability that the same die is used for the first three rolls? b. Suppose that we observe the first two rolls. What is p (z1 = 1 | x1 = 2, x2 = 4), the probability that the casino used the fair die in the first roll? c. Using the backward algorithm, compute the probability that we observe the sequence x1 = 2, x2 = 3, x3 = 3, x4 = 3 and x5 = 1. Show your work (i.e., show each of your belief for based on time). Consider the final distribution at time t = 6 for both p (z_t = 1) = p (z_t = 2) = 1.

ANSWER:

Let say we have that the first state of the die is state 1. Therefore the probability of this is p(z1=1)=0.5.

Also the probability that the same die is used(i.e. casino would be in the same state) is p(z2=1|z1=1)=0.8.

Again, suppose the first state of the die is state 2. So, p(z1=2)=0.5 and p(z2=2|z1=2)=0.9.

Other transition probabilities can be written as

p(zt+1=2|zt=1)=1-p(zt+1=1|zt=1)=.2

p(zt+1=1|zt=2)=1-p(zt+1=2|zt=2)=.1

p(z3=1|z1=1) = [p(z3=1|z2=2)*p(z2=2|z1=1)]+[p(z3=1|z2=1)*p(z2=1|z1=1)] = 0.1*0.2+0.8*0.8 = 0.66

p(z3=2|z1=2) = [p(z3=2|z2=2)*p(z2=2|z1=2)]+[p(z3=2|z2=1)*p(z2=1|z1=2)] = 0.9*0.9+0.2*0.1 = 0.83

With this, the total probability that the same die is used for the first three rolls (i.e. casino would be in the same state) is  given thus;

{p(z1=1)*p(z3=1|z1=1)}*{p(z1=2)*p(z3=2|z1=2)}

=  0.5*0.66+0.5*0.83 = 0.745

Prob = 0.745

Algebra Review

Write an algebraic expression for each verbal expression.

1. the sum of one-third of a number and 27

2. the product of a number squared and 4

3. Write a verbal expression for 5n^3 +9.

Answers

Answer:

Step-by-step explanation:

1. The sum of one-third of a number and 27

= [tex]\frac{1}{3}\times x +27\\= 1/3x +27[/tex]

2. The product of a number squared and 4

[tex]Let\:the\:unknown\: number\: be \:x\\\\x^2\times4\\\\= 4x^2[/tex]

3.Write a verbal expression for 5n^3 +9.

The sum of the product and of 5 and a cubed number and 9

Solve for x: 3(x + 1)= -2(x - 1) + 6.

Answers

Answer:

x=1

Step-by-step explanation:

3(x + 1)= -2(x - 1) + 6.

Distribute

3x+3 = -2x+2+6

Combine like terms

3x+3 = -2x+8

Add 2x to each side

3x+3+2x = 8

5x+3 = 8

Subtract 3 from each side

5x =5

Divide by 5

x =1

Find the largest number apart from 840 that is a multiple of 24 and a factor of 840.​

Answers

Answer:

168

Step-by-step explanation:

first, split both 840 and 24 into there primes.

840=2×2×2×3×5×7

24=2×2×2×3

therefore any multiples of 24 must have those factor.

so, factors of 840 that are multiples of 24 are

2×2×2×3=24

2×2×2×3×5=120

2×2×2×3×7=168

2×2×2×3×5×7=840

there the answer is 168

Find the product of the roots of the equation
xl-5x - 36 = 0

Answers

Answer:

Step-by-step explanation:

Hello, I assume that you mean

[tex]x^2-5x-36[/tex]

The product is -36.

[tex]x_1 \text{ and } x_2 \text{ are the two roots, we can write}\\\\(x-x_1)(x-x_2)=x^2-(x_1+x_2)x+x_1\cdot x_2[/tex]

So in this example, it means that the sum is 5 and the product is -36.

Thank you

(3) In a group of 60 seldiers have enough food for
20 days - How many soldiers should leave the group
so that the food is enough for 100 days ? Find it.​

Answers

Answer:

48 soldiers

Step-by-step explanation:

60 soldiers                    20 days

x soldiers                       100 days

60 x 20 = 1200 = 100x

Therefore x = 1200/100 = 12

60 - 12 = 48

Hope that helped!!! k

Answer:

30 Soldiers

Step-by-step explanation:

Given:

1) No of soldiers=60

No of days=20

2) No of days=100

No of soldiers=?

No of soldiers to leave=?

Solution:

Let us use the cross multiplication method.

Let x be the no of soldiers.

No of days                                     No of  soldiers

1) 20                                                     60

2) 100                                                    x

by cross multiplying,

20x=100 x 60

20x=600

x=600/20

x=30 soldiers

Therefore, No of soldiers to leave =60-30=30 soldiers

algebra and trigonometry difference

Answers

Answer:

Algebra deals with knowing the value of unknown variables and functional relationships, while trigonometry touches on triangles, sides and angles and the relationship between them.

Algebra is more on polynomial equations, x and y while trigonometry more on sine, cosine, tangent, and degrees.

Trigonometry is much more complicated than algebra but algebra has its uses in our daily lives, be it calculating distance from point to another or determining the volume of milk in a milk container.

Step-by-step explanation:

Answer:

Although both Algebra II and Trigonometry involve solving mathematical problems, Algebra II focuses on solving equations and inequalities while Trigonometry is the study of triangles and how sides are connected to angles.

hope this answer helps u

pls mark as brainliest .-.

Divide. Write the quotient in lowest terms. 3 3/4 ÷ 5/7

Answers

Rewrite 3 3/4 as an improper fraction

3 3/4 = 15/4

Now you have

15/5 / 5/7

When you divide fractions, change the division to multiplication and flip the second fraction over:

15/4 x 7/5

Now multiply the top numbers together and the bottom numbers together:

( 15 x 7) / (4 x 5) = 105/20

Write as a proper fraction:

105/20 = 5 1/4

Answer: 21/4 or 5 1/4

Explanation:
3 3/4 divide by 5/7
= 15/4 x 7/5
= 105/20
= 21/4 or 5 1/4

Find the distance between points P(5, 1) and Q(3, 4) to the nearest tenth.
3.6
5
9.4
13

Answers

Answer:

≈ 3.6

Step-by-step explanation:

Calculate the distance d using the distance formula

d = [tex]\sqrt{(x_{2}-x_{1})^2+(y_{2}-y_{1})^2 }[/tex]

with (x₁, y₁ ) = (P(5, 1) and (x₂, y₂ ) = Q(3, 4)

d = [tex]\sqrt{(3-5)^2+(4-1)^2}[/tex]

   = [tex]\sqrt{(-2)^2+3^2}[/tex]

   = [tex]\sqrt{4+9}[/tex]

    = [tex]\sqrt{13}[/tex] ≈ 3.6 ( to the nearest tenth )

Answer:

3.6

Step-by-step explanation:

Look above bru

Twelve apples cost $2.00. How much will 50 apples cost?

Answers

Answer:

$8.33

Step-by-step explanation:

[tex]Solve \:using \: proportion\\\\12\:apples = \$ 2\\50\:apples = \$ x\\Cross \: Multiply\\\\12x = 100\\\\\\\frac{12x}{12} = \frac{100}{12} \\\\x = \$ 8.333[/tex]

Answer:

About $8.33.

Step-by-step explanation:

Write a proportion. Make sure the values line up horizontally:

[tex]\frac{12\text{ apples}}{\$2} =\frac{50\text{ apples}}{\$x}[/tex]

Cross multiply:

[tex]100=12x\\x=25/3\approx\$8.33[/tex]

which rigid transformation would map triangle AQR to triangle AKP

Answers

Step-by-step explanation:

A rotation about point A a reflection across the line containing AR a reflection across the line containing AQ a rotation about point R

Answer:

A rotation about point A

Step-by-step explanation:

I am taking the test if it is wrong I will add a comment

Suppose we want to choose 6 colors, without replacement, from 14 distinct colors. (a) How many ways can this be done, if the order of the choices matters? (b) How many ways can this be done, if the order of the choices does not matter?

Answers

Answer:

(a) 2,162,160

(b) 3,003

Step-by-step explanation:

(a) order matters

You can choose from 14 for the first pick. Then you have 13 left for the second pick. Then you have 12 left for the third pick. Keep going until you have 9 left for the 6th pick. The number when order matters is:

total = 14 * 13 * 12 * 11 * 10 * 9 = 2,162,160

(b) Order does not matter

Start with the same number as above for picking 6 out of 14. Since order does not matter, we divide by the number of ways you can arrange 6 items.

Since there are 6! ways of arranging 6 items,

total = 2,162,160/6! = 3,003

The number of ways when the order matters are 121080960.

The number of ways when order does not matters are 3003.

Given,

Choose 6 colors, without replacement, from 14 distinct colors.

We have to find:

- How many ways can this be done, if the order of the choices matters.

- How many ways can this be done if the order of the choices does not matter.

What are permutation and combination?

We use permutation when the order of the arrangements matters.

It is given by:

[tex]^ nP_r[/tex] = n! / r!

We use combination when order does not matter.

It is given by:

[tex]^nC_{r}[/tex] = n! / r! (n-r)!

Find the number of ways when order matters.

We have,

n = 14 and r = 6

[tex]^{14}P_{6}[/tex]

= 14! / 6!

= (14 x 13 x 12 x 11 x 10 x 9 x 8 x 7 x 6!) / 6!

= 4 x 13 x 12 x 11 x 10 x 9 x 8 x 7

= 121080960

Find the number of ways when order does not matter.

We have,

n = 14 and r = 6

[tex]^{14}C_{6}[/tex]

= 14! / 6! 8!

= 14 x 13 x 12 x 11 x 10 x 9 / 6 x 5 x 4 x 3 x 2

= 7 x 13 x 11  x 3  

= 3003

Thus,

The number of ways when the order matters are 121080960.

The number of ways when order does not matters are 3003.

Learn more about combination here:

https://brainly.com/question/28134115

#SPJ2

Decide if the situation involves​ permutations, combinations, or neither. Explain your reasoning. Does the situation involve​ permutations, combinations, or​ neither? Choose the correct answer below. A. B. C. Neither. A line of people is neither an ordered arrangement of​ objects, nor a selection of objects from a group of objects.

Answers

Answer:

C. Neither

Step-by-step explanation:

The permutation is a selection of objects from a given sample in an ordered manner .

The combination is a selection of objects from a given sample irrespective of an order of arrangement.

The given line of people is neither an ordered arrangement of​ objects, nor a selection of objects from a group of objects So it fits neither of the combinations or permutations.

So the best answer is neither.

if P(x)=1+6x-5x^2 represents the profit in selling x thousand Boombotix speakers, how many speakers should be sold to maximize profit?

Answers

Answer:

600

Step-by-step explanation:

[tex]p(x) = 1 + 6x - 5x^2[/tex]

x max = [tex]-b/2a[/tex]

a = -5

b = 6

-6/2(-5) = 6/10 = 3/5 = .6

.6 thousand = 600

600 speakers should be sold.

Alternatively, you can check the vertex of the parabola formed.

The size of a television is the length of the diagonal of its screen in inches. The aspect ratio of the screens of older televisions is 4:3, while the aspect ratio of newer wide-screen televisions is 16:9. Find the width and height of an newer 75-inch television whose screen has an aspect ratio of 16:9

Answers

Answer:

The Width = 65.44 inches

The Height = 36.81 inches

Step-by-step explanation:

We are told in the question that:

The width and height of an newer 75-inch television whose screen has an aspect ratio of 16:9

Using Pythagoras Theorem we known that:

Width² + Height² = Diagonal²

Since we known that the size of a television is the length of the diagonal of its screen in inches.

Hence, for this new TV

Width² + Height² = 75²

We are given ratio: 16:9 as aspect ratio

Width = 16x

Height = 9x

(16x)² +(9x)² = 75²

= 256x² + 81x² = 75²

337x² = 5625

x² = 5625/337

x² = 16.691394659

x = √16.691394659

x = 4.0855103303

Approximately x = 4.09

For the newer 75 inch tv set

The Height = 9x

= 9 × 4.09

= 36.81 inches

The Width = 16x

= 16 × 4.09

= 65.44 inches.

I need help with these questions asap, I will post pictures if you know them all answer them in the order of the photos from 1-5 thank you.

Answers

Answer:

1. step 4

2.idk

3. step 2

4.-5n = 1 --------->  n= -1/5

n + 15 = -10 -------> -25

n/5 = -1/5 ------> n = -1

n - 13 = -12 ------> n = 1

5. cant see the drop down menu or possible answers

but if an answer is the addition one thing

then the second one is the subtraction thing

Step-by-step explanation:

The probability that a company will launch the product A and B are 0.45 and 0.60 respectively, in main while, probability that both products launched, is 0.35. what is the probability that Neither will of these products launch ? At least one product will be launched ?

Answers

Answer:

a) what is the probability that Neither will of these products launch ?

= 0.30

b) At least one product will be launched ?

= 0.70

Step-by-step explanation:

From the above question, we have the following information:

P(A) = 0.45

P(B) = 0.60

P(A ∩ B) = P(A and B) launching = 0.35

Step 1

We find the Probability that A or B will launch

P (A ∪ B) = P(A) + P(B) - P(A ∩ B)

= 0.60 + 0.45 - 0.35

= 1.05 - 0.35

= 0.70

a) what is the probability that Neither will of these products launch ?

1 - Probability ( A or B will launch)

= 1 - 0.70

= 0.30

b)At least one product will be launched?

This is equivalent to the probability that A or B will be launched

P (A ∪ B) = P(A) + P(B) - P(A ∩ B)

= 0.60 + 0.45 - 0.35

= 1.05 - 0.35

= 0.70

A random sample of 11 students produced the following data, where x is the hours spent per month playing games, and y is the final exam score (out of a maximum of 50 points). The data are presented below in the table of values.
x y
14 46
15 49
16 37
17 42
18 37
19 31
20 25
21 23
22 20
23 15
24 12
What is the value of the intercept of the regression line, b, rounded to one decimal place?

Answers

Answer:

b = - 3.7

Step-by-step explanation:

here are the data values:

x   y          XY        X²

14 46         644       196

15 49         735       225

16 37          592      256

17 42          714        289

18 37          666      324

19 31           589      361

20 25         500      400

21 23          483       441

22 20         440       484

23 15          345       529

24 12         288       576

now we are required to find the summation (total) of all values of X, Y, XY and X².

∑X = 209

∑Y = 337

∑XY = 5996

∑X² = 4081

The formular for finding b is given as:

b = n∑XY - (X)(Y) / n∑X² - (∑X)²

= 11(5996) - (209)(337) / 11(4081) - (209)²

= 65956 - 70433 / 44891 - 43681

= -4477/ 1210

= -3.7

The question asked us to find the value of b but we can  go further to find the equation of the regression line:

a = ∑Y - b∑X / n

= 337 - (-3.7)(209)/  11

=1110.3/11

= 100.94

the equation is:

Y = 100.94 - 3.7X

I hope you find my solution useful!

=

About how many feet are in 3.6 kilometers? 1 m = 39.37 in

Answers

Answer:

11811 feet

Step-by-step explanation:

Hope it helps!

There are about 11,812 feet in 3.6 kilometers.

To convert kilometers to feet, we need to use the conversion factor:

1 kilometer = 3,280.84 feet.

Now, to find how many feet are in 3.6 kilometers,

we can multiply 3.6 by the conversion factor:

So, 3.6 kilometers x 3,280.84 feet/kilometer

= 11,811.504 feet.

Thus, Rounded to a whole number, there are about 11,812 feet in 3.6 kilometers.

Learn more about Unit Conversion here:

https://brainly.com/question/14573907

#SPJ6

An article contained the following observations on degree of polymerization for paper specimens for which viscosity times concentration fell in a certain middle range:

418 421 421 422 425 428 431 435 437
438 445 447 448 453 458 462 465
(c) Calculate a two-sided 95% confidence interval for true average degree of polymerization. (Round your answers to two decimal places.) Note that it is plausible that the given sample observations were selected from a normal distribution and there are no outliers.

(___ , ___)



Does the interval suggest that 441 is a plausible value for true average degree of polymerization?

Yes or No


Does the interval suggest that 451 is a plausible value?

Yes or No

Answers

Answer:

Step-by-step explanation:

Form a set of values we get

n = 17

And with the help of a calculator

μ₀ = 438,47

σ  = 14,79

Normal Distribution is :  N ( 438,47 ; 14,79 )

c)

CI = 95 % means  α = 5 %    α/2 = 2,5 %    α/2 = 0,025

and as n < 30  we should use t-student distribution with n -1 degree of freedom  df = 16.  t score for 0,025 and 16 s from t-table 2,120

By definition:

CI = [  μ₀  ±  t α/2 ; n-1 * σ/√n ]

CI = [  μ₀  ± 2,120* 14,79/√17 ]

CI = [  μ₀  ±  7,60 ]

CI = [ 438,47 ± 7,60 ]

CI = [  430,87 ; 446,07 ]

95% confidence interval for true average degree of polymerization is  [430.87 ; 446.07] and this interval suggest that 441 is a plausible value for true average degree of polymerization and also this interval does not suggest that 451 is a plausible value.

Given :

Sample = [ 418, 421, 421, 422, 425, 428, 431, 435, 437,  438, 445, 447, 448, 453, 458, 462, 465 ]95% confidence interval.

The total number of values given is, n = 17

Mean, [tex]\mu_0=438.47[/tex]

Standard Deviation, [tex]\sigma = 14.79[/tex]

The normal distribution is given by: N (438.47 ; 14.79)

If Cl  is 95% then [tex]\alpha[/tex] is 5% and [tex]\alpha /2[/tex] is 2.5%

[tex]\alpha /2 = 0.025[/tex]

Now, use t-statistics distribution with (n-1) degree of freedom df = 16

So, the t score for 0.025 and 16 s from t-table 2.120.

[tex]\rm Cl = [\mu_0 \pm t_{\alpha /2};(n-1)\times \dfrac{\sigma}{\sqrt{n} }][/tex]

[tex]\rm Cl = [\mu_0 \pm 2.120\times \dfrac{14.79}{\sqrt{17} }][/tex]

[tex]\rm Cl = [\mu_0 \pm 7.60][/tex]

Cl = [430.87 ; 446.07]

Yes, the interval suggests that 441 is a plausible value for true average degree of polymerization.

No, the interval does not suggest that 451 is a plausible value.

For more information, refer to the link given below;

https://brainly.com/question/2561151

(-1, 4) and (-2, 2).

Answers

Answer:

Slope : 2

slope-intercept: y = 2x + 6

Point-slope (as asked): y - 4 = 2 (times) (x + 1)

standered: 2x - y = -6

Step-by-step explanation:

4 solid cubes were made out of the same material. All four have different side lengths: 6cm, 8cm, 10cm, and 12cm. How to distribute the cubes onto two plates of a scale so the scale is balanced? Answer: A= the cube with side length 6 cm, B= the cube with side length 8 cm, C= the cube with side length 10 cm, D= the cube with side length 12 cm. On one side of the scale : , on the other side of the scale

Answers

Answer: The cube with side length of 12cm is alone in one plate, the other 3 cubes are in the other plate.

Step-by-step explanation:

We have 4 cubes with side lengths of:

6cm, 8cm, 10cm and 12cm.

Now, some things you need to know:

If we want a scale to be balanced, then the mass in both plates must be the same.

The volume of a cube of side length L is:

V = L^3

And the mass of an object of density D, and volume V is:

M = D*V.

As all the cubes are of the same material, all of them have the same density, so the fact that we do not know the value of D actually does not matter here.

Then we want to forms two groups of cubes in such a way that the total volume in each plate is the same (or about the same), the volumes of the cubes are:

Cube of 6cm:

V = (6cm)^3 = 216cm^3

Cube of 8cm:

V = (8cm)^3 = 512cm^3

Cube of 10cm:

V = (10cm)^3 = 1000cm^3

cube of 12cm

V = (12cm)^3 =  1728cm^3

First, if we add the volumes of the first two cubes, we have:

V1 = 216cm^3 + 512cm^3 = 728cm^3

Now we can see that we add 1000cm^3 the volume will be equal to the volume of the larger cube, so here we can also add the cube with side length of 10cm

Then the volume of the 3 smaller cubes together is:

V1 = 216cm^3 + 512cm^3 + 1000cm^3 = 1728cm^3.

Then, if we want to have the same volume in each plate, then we need to have the 3 smaller cubes in one plate, and the larger cube in the other plate.

Using Normal Distribution, what is the area to the right of 0.72 under the
standard normal curve?

Answers

Answer: 0.2358

Step-by-step explanation:

Using Normal Distribution, under the  standard normal curve

The area to the right of z is given by P(Z>z)=1-P(Z<z)

So, the area to the right of z= 0.72 under the  standard normal curve would be:

P(Z>0.72)=1-P(z<0.72)

=1-0.7642   [By using p-value table]

= 0.2358

Hence, the area to the right of z= 0.72 under the  standard normal curve is 0.2358 .

A Markov chain has 3 possible states: A, B, and C. Every hour, it makes a transition to a different state. From state A, transitions to states B and C are equally likely. From state B, transitions to states A and C are equally likely. From state C, it always makes a transition to state A.

(a) If the initial distribution for states A, B, and C is P0 = ( 1/3 , 1/3 , 1/3 ), find the distribution of X2

(b) Find the steady state distribution by solving πP = π.

Answers

Answer:

A) distribution of x2 = ( 0.4167 0.25 0.3333 )

B) steady state distribution = [tex]\pi a \frac{4}{9} , \pi b \frac{2}{9} , \pi c \frac{3}{9}[/tex]

Step-by-step explanation:

Hello attached is the detailed solution for problems A and B

A) distribution states for A ,B, C:

Po = ( 1/3, 1/3, 1/3 )  we have to find the distribution of x2 as attached below

after solving the distribution

x 2 = ( 0.4167, 0.25, 0.3333 )

B ) finding the steady state distribution solving

[tex]\pi p = \pi[/tex]

below is the detailed solution and answers

Find all real solutions of the equation: x 2 + 3x − 10 = 0

Answers

Answer: x=8/3 or x= 2.6666....

Step-by-step explanation:

[tex]2+3x-10=0[/tex]

[tex]2-10=-8[/tex]

[tex]3x-8=0[/tex]

add 8 on both sides

[tex]3x-8+8=0+8[/tex]

[tex]3x=8[/tex]

divide 3 on both sides

[tex]x=\frac{8}{3}[/tex]

Answer:

8/3

Step-by-step explanation:

2 +3x + 10 = 0

2-10 +3x = 0

-8 + 3x = 0

3x = 8

x = 8/3

Other Questions
In regard to policy making, the "iron triangle" means __________. Group of answer choices the Department of Defense, the Department of State, and the Department of Justice that alliance of related interest groups, legislators in Congress, and bureaucrats a term for the constitutional barriers to regulation the three branches of government the CIA, the FBI, and the Bureau of Alcohol, Tobacco and Firearms (ATF) On March 15, 20X7, Barrel Company paid property taxes of $120,000 on its factory building for calendar year 20X7. On July 1, 20X7, Barrel made $20,000 in unanticipated repairs to its machinery. The repairs will benefit operations for the remainder of the calendar year. What total amount of these expenses should be included in Barrel's quarterly income statement for the three months ended September 30, 20X7? The narrator of 'The Leap" describes walking to a gravesite justdown the highway when she was growing up. Who is buried inthat gravesite?Athe unborn daughter whom her mother lost in the circus disasterBthe spectators who were lost in the circus disasterher mother's first husbandDher fatherHelp!Please The members of an Olympiad team contributeda total of $ 1.69 for refreshments for theirweekly practices. Each member contributed thesame amount and paid for his or her share infive coins. How many nickels were contributedby all of the members? Sagan scored 1200 on the SAT. The distribution of SAT scores in a reference population is normally distributed with mean 980 and standard deviation 100. Andrea scored 27 on the ACT. The distribution of ACT scores in a reference population is normally distributed with mean 20 and standard deviation 5. Who performed better on the standardized exams and why? Sagan scored higher than Andrea. Sagan's standardized score was 2.2, which is 2.2 standard deviations above the mean and Andrea's standardized score was 1.4, which is 1.4 standard deviations above the mean. Sagan scored higher than Andrea. Sagan's score was a 1200, which is greater than Andrea's score of 27. Andrea scored higher than Sagan. Andrea's standardized score was 1.4, which is 1.4 standard deviations above the mean, but closer to the mean than Sagan's standardized score of 2.2 standard deviations above the mean. Sagan scored higher than Andrea. Sagan's score was 220 points above the mean of 980, and Andrea's was 7 points above the mean of 20. Andrea scored higher than Sagan. Andrea is only 9 points from the top score of 36 on the ACT, and Sagan is 400 points from the top score of 1600 on the SAT. Ncluding a 6% sales tax, a new stereo costs $492.9. Find the cost of the stereo before tax. A) First write an equation you can use to answer this question. Use x x as your variable and express any percents in decimal form in the equation. (1) -3(-5x-2u+1) use the distributive property to remove the parentheses What season is it in St. Petersburg, Russia on March 30th? When Mr. Gree bought a used car he made adown payment of $825. This was 30% of thetotal cost. The total cost was:PLEASE HELP! QUICKLY PLEASE! f(x)=3x2+10x-25 g(x)=9x2-25 Find (f/g)(x). Four couples are at a party. Four of the eight people are randomly selected to win a prize. No person can win more than one prize. What is the probability that both of the members of at least one couple win prizes? Express your answer as common fraction. Niko is 3 times as old as Lila. Niko's age is the same as adding Lila's age to the product of 3 and Amber's age. Niko is 45 years old. Kameron's age is equal to 2 times the sum of Amber's age and Lila's age. How old is Kameron? years old The sum of 6 even integers is 130 what is the 2nd number in the sequence What shape best describes the cross-section cut at an angle to the base of a right rectangular prism? Trapezoid Parallelogram Square Rectangle PLEASE help me solve this question! No nonsense answers please! Simplify: 1/2 (2a + b) - (4a + b) Show ALL work please! 2-last year, the weather this summer is warmer and drier.a. In comparison with b. However c. Instead of d. whereas You are interested in buying a piece of land overlooking the sea. You find a place atop a 50m high sea cliff. The lot is only about 30m wide but is 200m long parallel to the coast, so you think you are getting a large expanse of the coastline. What would be the most important thing to look for in considering this purchase a person Travels along a straight road for half the distance with velocity V1 and the remaining half the distance with velocity V2 the average velocity is given by The numbers of words defined on randomly selected pages from a dictionary are shown below. Find the mean, median, mode of the listed numbers. 72 58 62 38 44 66 42 49 76 52 What is the mean? Select the correct choice below and ,if necessary ,fill in the answer box within your choice.(around to one decimal place as needed)